www.vorhilfe.de
Vorhilfe

Kostenlose Kommunikationsplattform für gegenseitige Hilfestellungen.
Hallo Gast!einloggen | registrieren ]
Startseite · Forum · Wissen · Kurse · Mitglieder · Team · Impressum
Forenbaum
^ Forenbaum
Status Englisch
  Status Grammatik
  Status Lektüre
  Status Korrekturlesen
  Status Übersetzung
  Status Sonstiges (Englisch)

Gezeigt werden alle Foren bis zur Tiefe 2

Navigation
 Startseite...
 Neuerdings beta neu
 Forum...
 vorwissen...
 vorkurse...
 Werkzeuge...
 Nachhilfevermittlung beta...
 Online-Spiele beta
 Suchen
 Verein...
 Impressum
Das Projekt
Server und Internetanbindung werden durch Spenden finanziert.
Organisiert wird das Projekt von unserem Koordinatorenteam.
Hunderte Mitglieder helfen ehrenamtlich in unseren moderierten Foren.
Anbieter der Seite ist der gemeinnützige Verein "Vorhilfe.de e.V.".
Partnerseiten
Weitere Fächer:

Open Source FunktionenplotterFunkyPlot: Kostenloser und quelloffener Funktionenplotter für Linux und andere Betriebssysteme
Forum "Funktionalanalysis" - Bild stetiger Operatoren
Bild stetiger Operatoren < Funktionalanalysis < Analysis < Hochschule < Mathe < Vorhilfe
Ansicht: [ geschachtelt ] | ^ Forum "Funktionalanalysis"  | ^^ Alle Foren  | ^ Forenbaum  | Materialien

Bild stetiger Operatoren: Beweis,
Status: (Frage) beantwortet Status 
Datum: 16:07 So 23.01.2011
Autor: Balendilin

Aufgabe
Gegeben ist ein linearer Operator T: [mm] X\rightarrow [/mm] Y, wobei X und Y normierte Räume (mit den Normen [mm] ||\cdot||_x [/mm] bzw. [mm] ||\cdot||_Y) [/mm] über dem selben Körper sind.

zu zeigen ist, dass folgende Aussagen äquivalent sind:

(1) T ist stetig
(2) T ist beschränkt
(3) Das Bild unter T der offenen oder abgeschlossenen Einheitskugel in X ist beschränkt in Y
(4) Das Bild unter T jeder in X beschränkten Menge ist beschränkt in Y
(5) Es gibt eine Kugel um 0 in X, deren Bild in Y beschränkt ist

Einige Richtungen habe ich schon bewiesen.
1 [mm] \Leftrightarrow [/mm] 2 habe ich hin bekommen

Aus 2 folgt dann 3:
[mm] ||x||_x \leq [/mm] 1 [mm] \Rightarrow ||Tx||_y\leq c\cdot||x||_x=c [/mm] (da T beschränkt ist)

Genau so folgt auch [mm] 2\Rightarrow [/mm] 4 und [mm] 2\Rightarrow [/mm] 5

Ist folgender Beweis für [mm] 5\Rightarrow [/mm] 1 ok:
Sei U diese Kugel. T|U (T eingeschränkt auf U) ist also beschränkt. Damit ist T|U stetig auf U, insbesondere also in einem Pkt. (z.B. der 0). Und damit ist auch T in 0 stetig und damit auf ganz X
Der Beweis für [mm] 3\Rightarrow [/mm] 1 wäre analog.

Bekommt man irgendwie einen Beweis für [mm] 5\Rightarrow [/mm] 4 oder 3 [mm] \Rightarrow [/mm] 4 hin?

        
Bezug
Bild stetiger Operatoren: Antwort
Status: (Antwort) fertig Status 
Datum: 07:29 Mo 24.01.2011
Autor: fred97


> Gegeben ist ein linearer Operator T: [mm]X\rightarrow[/mm] Y, wobei
> X und Y normierte Räume (mit den Normen [mm]||\cdot||_x[/mm] bzw.
> [mm]||\cdot||_Y)[/mm] über dem selben Körper sind.
>  
> zu zeigen ist, dass folgende Aussagen äquivalent sind:
>  
> (1) T ist stetig
>  (2) T ist beschränkt
>  (3) Das Bild unter T der offenen oder abgeschlossenen
> Einheitskugel in X ist beschränkt in Y
>  (4) Das Bild unter T jeder in X beschränkten Menge ist
> beschränkt in Y
>  (5) Es gibt eine Kugel um 0 in X, deren Bild in Y
> beschränkt ist
>  Einige Richtungen habe ich schon bewiesen.
> 1 [mm]\Leftrightarrow[/mm] 2 habe ich hin bekommen
>  
> Aus 2 folgt dann 3:
>  [mm]||x||_x \leq[/mm] 1 [mm]\Rightarrow ||Tx||_y\leq c\cdot||x||_x=c[/mm]
> (da T beschränkt ist)
>  
> Genau so folgt auch [mm]2\Rightarrow[/mm] 4 und [mm]2\Rightarrow[/mm] 5
>  
> Ist folgender Beweis für [mm]5\Rightarrow[/mm] 1 ok:
>  Sei U diese Kugel. T|U (T eingeschränkt auf U) ist also
> beschränkt.


Das hast Du falsch verstanden.  Es bedeutet:  T(U) ist beschränkt, d.h. : es gibt ein c>0 mit:

                      $||Tx|| [mm] \le [/mm] c$   für jedes x [mm] \in [/mm] U


FRED

> Damit ist T|U stetig auf U, insbesondere also
> in einem Pkt. (z.B. der 0). Und damit ist auch T in 0
> stetig und damit auf ganz X
>  Der Beweis für [mm]3\Rightarrow[/mm] 1 wäre analog.
>  
> Bekommt man irgendwie einen Beweis für [mm]5\Rightarrow[/mm] 4 oder
> 3 [mm]\Rightarrow[/mm] 4 hin?


Bezug
Ansicht: [ geschachtelt ] | ^ Forum "Funktionalanalysis"  | ^^ Alle Foren  | ^ Forenbaum  | Materialien


^ Seitenanfang ^
www.englischraum.de
[ Startseite | Forum | Wissen | Kurse | Mitglieder | Team | Impressum ]